ChaseDream

标题: TTGWD25-3 帮帮忙!!! [打印本页]

作者: benman    时间: 2006-8-23 20:19
标题: TTGWD25-3 帮帮忙!!!

Mel: The official salary for judges has always been too low to attract the best

Candidates to the job. The legislature’s move to raise the salary has

done nothing to improve the situation, because it was coupled with

a ban on receiving money for lectures and teaching engagements.

Pat:  No, the raise in salary really does improve the situation. Since very few

judges teach or give lectures, the ban will have little or no negative

effect.

Pat’s response to Mel is inadequate in that it

A.    attempts to assess how a certain change will affect potential members

of a group by providing evidence about its effect on the current members.

B.     mistakenly takes the cause of a certain change to be an effect of that change

C.     attempts to argue that a certain change will have a positive effect merely

by pointing to the absence of negative effects

D.    simply denies Mel’s claim without putting forward any evidence in support

of that denial

E.     assumes that changes that benefit the most able members of a group

necessarily benefit all members of that group.

 

the answer offered is A but I choose C. Why??? Thanks...

                                  
作者: imp01    时间: 2006-8-23 21:18

正是因为BAN,很多best  Candidates 才不去做JUDGES的

而不能说是因JUDGES教书的不多,BAN 也就无所谓


作者: benman    时间: 2006-8-23 22:10
以下是引用imp01在2006-8-23 21:18:00的发言:

正是因为BAN,很多best  Candidates 才不去做JUDGES的

而不能说是因JUDGES教书的不多,BAN 也就无所谓

yeah, you are right. thx

Then what's wrong with C? Isn't Chioce C just the same logical structure as Pat's response???


作者: caterpillarcn    时间: 2006-8-23 23:27

I think we have to take into consideration the argument Mel gave: raising the pay for the judge will have positive effect if there is no ban. so disproving the ban's negative effect does show that the tactic will have a positive effect.
so there is nothing wrong with the logic of choice C.

I think we have to take into consideration the argument Mel gave: raising the pay for the judge will have positive effect if there is no ban. so disproving the ban's negative effect does show that the tactic will have a positive effect.
so there is nothing wrong with the logic of choice C.


[此贴子已经被作者于2006-8-23 23:29:37编辑过]

作者: 当当84    时间: 2006-8-25 10:54

谁能帮忙解释一下,这道题,真的不知道c错在哪里?


作者: caterpillarcn    时间: 2006-8-25 11:28
C is correct in its logic, so C is not inadequate.

作者: zzlz    时间: 2007-1-7 14:59
以下是引用caterpillarcn在2006-8-25 11:28:00的发言:
C is correct in its logic, so C is not inadequate.

这么解释吗?

C表达的意思是通过指出没有负面影响来支持一个有正面影响的观点.显然不是原文的逻辑矛盾所在,所以错误.

这道逻辑题的冲突点是把一个导致的现象,也就是结果描述成为原因.

mel:因为禁令不能教书和上课而导致收入少

pat:老师很少上课和教书啊.(废话)


作者: freshfruit    时间: 2007-1-7 19:38

是选择更好的,

C只是概括的题目中表达的特殊场景,而A是对场景本质的一般性概括。

A显然比C更适合选


作者: leoozop    时间: 2007-1-12 15:27
以下是引用caterpillarcn在2006-8-23 23:27:00的发言:

I think we have to take into consideration the argument Mel gave: raising the pay for the judge will have positive effect if there is no ban. so disproving the ban's negative effect does show that the tactic will have a positive effect.
so there is nothing wrong with the logic of choice C.

I think we have to take into consideration the argument Mel gave: raising the pay for the judge will have positive effect if there is no ban. so disproving the ban's negative effect does show that the tactic will have a positive effect.
so there is nothing wrong with the logic of choice C.

I think we have to take into consideration the argument Mel gave: raising the pay for the judge will have positive effect if there is no ban. so disproving the ban's negative effect does show that the tactic will have a positive effect.
so there is nothing wrong with the logic of choice C.


支持A,这题是围绕加工资的有效性来说的,不是BAN的有效性~

M加工资没作用,因为BAN禁止讲座。

F加工资有作用,因为现在很少J去讲座,BAN将来没有消极作用。

C讲一个变化的积极作用通过说它没有消极作用。

这里的变化是指的要不要加工资,而不是要不要取消BAN。


作者: 小蓓    时间: 2007-3-5 20:57

A.     attempts to assess how a certain change will affect potential members

of a group by providing evidence about its effect on the current members.

看了又看,感觉A中的potential members有问题,不知指什么?

大家选A的理由是:把BAN已造成的影响误作BAN之前的状况。可是A说的是把对current members的影响当作对potential members的影响

支持C


作者: tidusyuna    时间: 2007-9-9 22:08

应该是C.

Mel的论述中,结论明显是

The legislature’s move to raise the salary has

done nothing to improve the situation

Pat的response:No, the raise in salary really does improve the situation.也正是P的conclusion.后面是premise.

所以c项attempts to argue that a certain change will have a positive effect,正是它论点所在,而merely by pointing to the absence of negative effects是指出由premise到conlusion的推理错误.

A项重心明显是关于premise的,而response 应该是对应于conclusion的.

 

 

The legislature’s move to raise the salary has

done nothing to improve the situation

Pat的response:No, the raise in salary really does improve the situation.也正是P的conclusion.后面是premise.

所以c项attempts to argue that a certain change will have a positive effect,正是它论点所在,而merely by pointing to the absence of negative effects是指出由premise到conlusion的推理错误.

A项重心明显是关于premise的,而response 应该是对应于conclusion的.

 

 


作者: wsdoll    时间: 2007-9-23 09:15
up...
作者: riverquiet    时间: 2007-10-7 04:18
I]:----(
[此贴子已经被作者于2007-10-7 4:19:02编辑过]

作者: sch    时间: 2007-12-14 13:36
C不对,通过指出没有坏处,来证明有好处.哪里对哪里


其实主要错误是,正是因为ban了,所以才没有人教书讲座开班讲课,用了结果当作原因.

找出描述这种错误的选项

作者: weideng    时间: 2008-3-31 16:29
可以用AWA的思路,如果这道是AA题,大家一定能想到all things are equal的错误吧
作者: lichabrend    时间: 2008-5-27 15:49
up
作者: yzhao26    时间: 2008-6-8 10:02
up
作者: MarilynR    时间: 2008-6-12 11:42

本来我也选的C,现在想通了选A

C说通过说明没有负面影响而证明该变化有正面影响à文章没有说
  
没有负面影响
  
这一点。

A中的Potential member 指的是现在不是judge,但有可能因为工资待遇调高而成为judge的人。(这些人很有可能在大学教书)

Pat只是说明了现在的judge里很少人教书,但这并不意味着potential member不会想要教书。搞不好人家教习惯了会很想keep it as a part time job.

Open to discussion.


[此贴子已经被作者于2008-6-12 11:43:49编辑过]

作者: bigzigzag    时间: 2008-6-14 06:26

"the ban will have little or no negative effect."  =    by pointing to the absence of negative effects

In pat's argument , " Since very few judges teach or give lectures, the ban will have little or no negative effect. " is the cause

"No, the raise in salary really does improve the situation." is the result.

that is not correct.


作者: 夜凉如水    时间: 2008-8-2 21:42
以下是引用MarilynR在2008-6-12 11:42:00的发言:

本来我也选的C,现在想通了选A

C说通过说明没有负面影响而证明该变化有正面影响à文章没有说
  
没有负面影响
  
这一点。

A中的Potential member 指的是现在不是judge,但有可能因为工资待遇调高而成为judge的人。(这些人很有可能在大学教书)

Pat只是说明了现在的judge里很少人教书,但这并不意味着potential member不会想要教书。搞不好人家教习惯了会很想keep it as a part time job.

Open to discussion.


嗯想通了 谢谢
作者: jonathan1987    时间: 2008-12-9 20:22

bbbbbbbbbbbbbbbb


作者: jean1280    时间: 2009-5-25 10:30
UP
作者: jean1280    时间: 2009-7-3 20:46
UP
作者: nancia    时间: 2009-9-2 13:22
看看
[此贴子已经被作者于2009/9/2 13:54:46编辑过]

作者: fannybeibei    时间: 2010-8-5 21:52
以下是引用caterpillarcn在2006-8-25 11:28:00的发言:
C is correct in its logic, so C is not inadequate.
这么解释吗?
C表达的意思是通过指出没有负面影响来支持一个有正面影响的观点.显然不是原文的逻辑矛盾所在,所以错误.
这道逻辑题的冲突点是把一个导致的现象,也就是结果描述成为原因.
mel:因为禁令不能教书和上课而导致收入少
pat:老师很少上课和教书啊.(废话)


-- by 会员 zzlz (2007/1/7 14:59:00)





那不就是B嘛!
为什么。。。
作者: wallstreeter    时间: 2011-6-11 11:39
想通了,选A
Key Point:change只有raise the salary和ban
Pat的逻辑:
1、因为现在judge里面教书的少,所以change以后ban影响小
2、因为ban的影响小,所以change会改善情况
A指出了1中的逻辑问题在于all things are equal
C说的是2有逻辑错误,但是2真的有逻辑错误么?
如果ban的影响小,由于change只有raise the salary和ban,就是把唯一的不利改善的影响排除,完全可以得到结论:change会改善情况
所以2中是没有逻辑错误的。
作者: 淡云清风    时间: 2011-8-10 17:49
我用排除法:
B. 涨薪的因果颠倒——实际上,M说涨薪的原因是因为薪水太低,不能吸引人,而P并没有提到这点,所以不能说因果颠倒。
C. 有正面效果的原因仅仅是因为没有负面效果——而原文中P有说have little or no negative effect,说明P承认是有可能有负面效果的,而不是完全没有
D. 没有进一步的证据——无关选项
E. 假设使大部分成员受益的改变(涨薪)会是全体成员受益——P并没说这点,只说了认为会改善现状。

再反观A. 打算用一个会影响现在成员的改变来判断此改变也会影响潜在成员(即将来的成员)——没有任何与原文不符的成分或因素。




欢迎光临 ChaseDream (https://forum.chasedream.com/) Powered by Discuz! X3.3